You are on page 1of 6

Esercizi Svolti durante il Corso

Es. 1. Data la funzione


f (x, y) = x2 − y2
(a) Calcolare la derivata direzionale di f nel punto (1, 2) lungo la dire-
1 1
zione individuata dal versore v = ( √ , √ )
2 2
(b) Determinare il minimo e il massimo assoluti di f nella regione D
1 1 1 1
delimitata dal trapezio di vertici (1, 2),(−1, 2),( , ) e (− , ),
4 2 4 2
compresi i lati del trapezio
Soluzione
(a) La funzione f è di classe C1 in R2 e quindi è differenziabile in ogni
punto del piano.
fx (x, y) = 2x
fy (x, y) = −2y
fx (1, 2) = 2
fy (1, 2) = −4
∂f 1 1 √
(1, 2) = 2 √ − 4 √ = − 2
∂v 2 2
(b) Si osservi che ∇ f (x, y) = (0, 0) se e solo se x = y = 0 e (0, 0) ∈
/ D.
Pertanto occorre stdiare la funzione sui lati del trapezio:
1 1 1 1
f (x, ) = x2 − , − ≤x≤
2 4 4 4
1
f (x, 2x) = −3x2 , − ≤x≤1
4
2
f (x, 2) = x − 4, −1 ≤ x ≤ 1
1
f (x, −2x) = −3x2 , −1 ≤ x ≤ −
4
e di conseguenza il minimo e il massimo sono necessariamente tra
i seguenti valori:
1 1
f (0, ) = −
2 4
f (0, 2) = −4
f (1, 2) = f (−1, 2) = −3
1 1 1 1 3
f ( , ) = f (− , ) = − (1)
4 2 4 2 16

1
Come si vede in (1)
3
In conclusione, min f = −4 e max f = −
D D 16

Es. 2. Data la funzione f (x, y) = x − y determinare massimi e mini-


mi nell’insieme A = {(x, y) ∈ R2 : −1 ≤ x ≤ 1, −2 ≤ y ≤ 2}

Soluzione
La funzone f è continua in A, insieme chiuso e lmitato di R2 , e quindi
per il teorema di Weierstrass f , ha minimo assoluto e massimo assoluto
in A.
La funzione f non ha punti stazionari. Dallo studio di f sui lati del
rettangolo segue che il massimo assoluto è f (1, −2) = 3 e il minimo as-
soluto è f (−1, 2) = −3

Es. 3. Data la funzione f (x, y) = x2 − y2 + 2xy determinare massimi


e minimi nell’insieme A = {(x, y) ∈ R2 : x2 + y2 ≤ 1}

Soluzione
La funzone f è continua in A, insieme chiuso e lmitato di R2 , e quindi
per il teorema di Weierstrass f , ha minimo assoluto e massimo assoluto
in A.
Poichè ∇ f (x, y) = (2x + 2y, −2y + 2x), l’unico punto stazionario di f
è (0, 0). Studiando la funzione f sulla circonferenza di centro (0, 0) e
raggio 1 si ha

g(θ ) = f (cos θ , sin θ ) = cos(2θ ), θ ∈ [0, 2π]


π π
g0 (θ ) = −2 sin(2θ ) + 2 cos(2θ ) = 0 ⇔ tan(2θ ) = 1, θ = + k , k = 0, 1, 2, 3...
8 2
π π 9π 9π
In conclusione, il massimo assoluto è f (cos , sin ) = f (cos , sin ) =
8 8 8 8
√ 5π 5π 13π 13π
2, mentre il minimo assoluto è f (cos , sin ) = f (cos , sin )=
√ 8 8 8 8
− 2

2
Es. 4. Minimizzare la funzione f (x) soggetta al vincolo g(x), dove:
min f (x1 , x2 ) = (x1 − 1)2 + (x2 − 2)2
g(x) = x1 + x2 − 2 ≤ 0
Soluzione
Possiamo intodurre la funzione penalizzata
Fk (x) = f (x) + k(g+ (x))2 = (x1 − 1)2 + (x2 − 2)2 + k(max((x1 + x2 − 2), 0))
Calcoliamo i punti stazionari:
∂ Fk
= 2(x1 − 1) + 2k(max((x1 + x2 − 2), 0)) = 0
∂ x1
∂ Fk
= 2(x2 − 2) + 2k(max((x1 + x2 − 2), 0)) = 0
∂ x2
Dai quali ottengo che:
k+1
x1,k =
2k + 1
3k + 2
x2,k =
2k + 1
Facendo tendere k → ∞ si ha:
1
x1 =
2
3
x2 =
2
Ora possiamo introdurre la seguente funzione penalizzata (la stessa fun-
k
zione del teorema di Fritz-John, con invece di k)
2
k + k
Fk (x) = f (x) + (g (x))2 = (x1 − 1)2 + (x2 − 2)2 + (max((x1 + x2 − 2), 0))
2 2
Calcoliamo i punti stazionari:
∂ Fk
= 2(x1 − 1) + k(max((x1 + x2 − 2), 0)) = 0
∂ x1
∂ Fk
= 2(x2 − 2) + k(max((x1 + x2 − 2), 0)) = 0
∂ x2
Dai quali ottengo che:
k
+1
x1,k = 2
k+1
3k
+2
x2,k = 2
k+1

3
Facendo tendere k → ∞ si ha:
1
x1 =
2
3
x2 =
2
Inoltre si ha che per k → ∞
kg+ (xk )
λ1k = 1 k →1 (2)
L
Es 5. Classificare gli eventuali punti stazionari della funzione defi-
nita in R2 da
f (x, y) = cos(x) + sin(y)
Soluzione
I punti stazionari della funzione sono dati dal seguente sistema

∂ 
 ∂ x f (x, y) = 0  x = kπ
 
 − sin(x) = 0

 
⇐⇒ ⇐⇒

 ∂  cos(y) = 0  y = π + jπ


 f (x, y) = 0 2
∂y
π
Si hanno dunque infiniti punti stazionari, ciascuno di coordinate (kπ, +
2
jπ), al variare di k ∈ Z, j ∈ Z. La matrice hessiana di un generico punto
(x, y) è  
− cos(x) 0
H(x, y) =
o − sin(y)
Calcolandola in un generico punto stazionario diventa
(−1)k+1
 
π 0
H(kπ, + jπ) =
2 o (−1) j+1
Il determinante della matrice Hessiana in un generico punto stazionario
risulta (−1) j+k . Dunque, se k e j sono entrambi pari o entrambi dispari,
il determinante risulta pari ad 1 e dunque positivo; si tratterà quindi di
un massimo o minimo relativo, a seconda del segno di (−1) j+k : se k è
pari si tratta di un punto di massimo relativo, se k è dispari si tratta di
un minimo relativo. Se k e j sono entrambi pari si tratta di un massimo
relativo. Se invece k e j sono entrambi dispari si tratta di un minimo
relativo. Infine se k è pari e j è dispari (o viceversa), si ha un punto di
sella poichè il valore del determinate è pari a −1.

4
Es 6. Data la funzione f (x, y) = x3 + y3 − (1 + x + y)3 in R2 , calco-
larne eventuali massimi e minimi relativi. Soluzione
Si calcolino i punti stazionari della funzione

∂f 2 2
 ∂ x = 3x − 3(1 + x + y) = 0


 ∂f
= 3y2 − 3(1 + x + y)2 = 0



∂y
Riscrivendo il sistema, sostituendo alla prima equazione la differenza tra
la prima e la seconda si ha:
 
 x 2 − y2 = 0  (x − y)(x + y) = 0
=⇒
 2
y − (1 + x + y)2
 2
y − (1 + x + y)2
Da cui otteniamo due sottosistemi
 
 x = −y  x=y

y2 − (1 + x + y)2 y2 − (1 + x + y)2
 

Per il primo sottosistema si può scrivere


 
 x = −y  x = −y
=⇒
y2 − (1 − y + y)2 = 0 y = ±1
 

si ottengono cosı̀ i punti A(1, −1) e B(−1, 1) Passando al secondo sotto-


sistema si ha che
 
 x=y  x=y
=⇒
 2
y − (1 + 2y)2 = 0
 2
3y + 4y + 1 = 0
 
−1 −1
si ottengono cosı̀ i punti C(−1, −1) e D , Si calcoli ora la
3 3
matrice Hessiana:
∂2 f
= 6x − 6(1 + x + y)
∂ x2
∂2 f
= 6y − 6(1 + x + y)
∂ y2
∂2 f
= −6(1 + x + y)
∂ x∂ y

5
Quindi la matrice Hessiana risulta essere
 2
∂2 f

∂ f
 ∂ x2 ∂ x∂ y 
 
 ∂2 f ∂2 f 
∂ x∂ y ∂ y2
Per A(1, −1) il determinate della matrice risulta essere

6 − 6 + (1 + 1 − 1) −6(1 + 1 − 1)
= −36 < 0
−6 6 − 6(1 + 1 − 1)
il punto è quindi di sella.
Per B(−1, 1) il determinate della matrice risulta essere

−12 −6
−6 0 = −36 < 0

il punto è quindi di sella.


Per C(−1, −1) il determinate della matrice risulta essere

0 6
6 0 = −36 < 0

il punto
 è quindidi sella.
−1 −1
Per D , il determinate della matrice risulta essere
3 3

−4 2
2 −4 = 12 > 0

∂2 f
Poichè il determinante della matrice è positivo e 2 (D) = −4 < 0, D è
∂x
un punto di massimo relativo.

You might also like